GYN Quiz 2

Pataasin ang iyong marka sa homework at exams ngayon gamit ang Quizwiz!

A patient presents with fever, leukocytosis, purulent vaginal discharge, and dyspareunia. On ultrasound, the uterus was ill-defined and the endometrium appeared irregular and thickened. What is the most likely diagnosis? A) Hydrosalpinx B) Pelvic inflammatory disease C) Endometrial hyperplasia D) Tubo-ovarian abscess

Pelvic inflammatory disease

A sonogram of a postpubertal woman demonstrated a 5-cm simple cyst on the left ovary. A follow-up exam 3 months later demonstrates a normal ovary. What do these findings likely represent? A) Resolution of corpus luteum B) Resolution of endometriosis C) Resolution of a follicular retention cyst D) Resolution of a hemorrhagic cyst

Resolution of a follicular retention cyst

A 25-year-old patient presented with hirsutism and abnormal periods. Sonography demonstrated a solid, complex ovarian mass. What is the most likely diagnosis? A) Leiomyosarcoma B) Sertoli-Leydig cell tumor C) Thecoma D) Dysgerminoma

Sertoli-Leydig cell tumor

Which of the following most accurately describes an endometrioma? A) overgrowth of endometrial tissue B) a collection of ectopic endometrial tissue C) ectopic location of active endometrial tissue D) ectopic endometrial tissue within the myometrium

a collection of ectopic endometrial tissue

Indications for saline infusion sonohysterography do not include A) active PID B) infertility C) Asherman's syndrome D) suspected septate uterus

active PID

Polycystic ovarian disease can result from: A) high levels of hCG B) unopposed estrogen C) an endocrine imbalance D) follicular hyperstimulation

an endocrine imbalance

A patient presents with a history of irregular menses. A transabdominal sonogram of the uterus demonstrates: A) endometritis B) Asherman syndrome C) endometrial hyperplasia D) an intrauterine contraceptive device

an intrauterine contraceptive device

An asymptomatic postmenopausal patient displays a 3.0-cm simple ovarian cyst. This finding is considered: A) rare B) benign C) benign D) malignant

benign

Which of the following best describes the sonographic appearance of uterine synechiae? A) thick, irregular endometrium B) hypoechoic endometrial mass C) irregular hypoechoic myometrial masses D) bright band of echoes within the endometrium

bright band of echoes within the endometrium

A common symptom of endometriosis is: A) amenorrhea B) menorrhagia C) dysmenorrhea D) urinary frequency

dysmenorrhea

Monitoring of which hormone is routine during ovarian induction therapy? A) estrogen B) estradiol C) progesterone D) follicle-stimulating hormone

estradiol

Days 6-13 in the ovary are which ovarian phase? A) proliferative B) secretory C) follicular D) luteal

follicular

Which fertility assistance program inserts oocytes and sperm into the fallopian tube? A) in vitro fertilization B) zygote intrafallopian transfer C) gamete intrafollicular transfer D) oocyte and sperm fallopian transfer

gamete intrafollicular transfer

A hormone used to trigger ovulation is: A) hCG B) Pergonal C) Falastem D) FSH E) Clomid

hCG

While scanning a 14-year-old patient who has not yet reached menarche, a distended vagina is seen, filled with low-level echoes. This sonographic appearance most likely represents ________, secondary to _______ A) urine filling vagina; pubertal incontinence B) pyocolpos; infection C) hematocolpos; imperforate hymen D) Gartner's duct cyst; residual Wollfian duct remnants

hematocolpos; imperforate hymen

Which of the following hormones stimulates ovulation? A) estrogen B) progesterone C) luteinizing hormone D) follicle-stimulating hormone

luteinizing hormone

The beginning of menses is known as: A) puberty B) menarche C) menopause D) amenorrhea

menarche

A patient complains of heavy menstrual cycles. This is most consistent with: A) menoxenia B) dyspareunia C) menorrhagia D) dysmenorrhea

menorrhagia

The endometrium appears thickest during the ______ portion of the menstrual cycle A) proliferative B) secretory C) luteal D) follicular

secretory

A patient presents with a history of a leiomyoma. Which location will most likely cause infertility? A) serosal B) subserosal C) intramural D) submucosal

submucosal

Which of the following fibroid locations is most likely to cause menorrhagia? A) cornual B) intramural C) subserosal D) submucosal

submucosal

If a patient displays an endometrial thickness of 2.0 cm, it is considered: A) suspicious for adenomyosis B) within normal limits in a menarche patient C) suspicious for proliferation of the endometrium D) within normal limits regardless of menstrual status

suspicious for proliferation of the endometrium

Which of the following describes the expected appearance of the endometrium in a patient using oral contraceptives? A) thin echogenic line B) thin hypoechoic line C) triple-line appearance D) thick and hyperechoic

thin echogenic line

The sonographic appearance of this endometrium is termed: A) shotgun sign B) decidual reaction C) triple-line pattern D) double decidua sign

triple-line pattern

What days of the menstrual cycle correlate with the menstrual phase? A) 1 to 5 B) 1 to 14 C) 15 to 21 D) 22 to 28

1 to 5

During the mid luteal phase, full luteal function is expected if the endometrial thickness is at least: A) 4 mm B) 8 mm C) 11 mm D) 14 mm

11 mm

Infertility is suggested when conception does not occur within: A) 6 months B) 9 months C) 12 months D) 24 months

12 months

If a patient relates a history of normal menses every 28 days, on which day of her cycle should ovulation occur? A) 7 B) 14 C) 21 D) 28 E) 30

14

Ovulation usually occurs when the dominant follicle reaches the following size: A) 10 mm B) 15 mm C) 1.5 cm D) 2.5 cm E) 3.5 cm

2.5 cm

Which of the following patients has an increased risk for developing endometrial hyperplasia? A) Patient with early menarche B) Breast cancer patient taking tamoxifen C) Pregnant patient D) Patient with yolk sac tumor

Breast cancer patient taking tamoxifen

Patients undergoing gamete intrafallopian tube transfer have increased risk for which of the following? A) Endometrial hyperplasia B) Ectopic pregnancy C) Asherman syndrome D) Fitz-Hugh-Curtis syndrome

Ectopic pregnancy

Perihepatitis can be associated with pelvic inflammatory disease, causing right upper quadrant tenderness and pain. This condition is: A) PD B) Fitz-Hugh Curtis syndrome C) Stein-Leventhal syndrome D) Indistinct uterus E) Meigs' syndrome

Fitz-Hugh Curtis syndrome

What layer of the endometrium is shed every month during menstruation? A) Perimetrium B) Basal layer C) Functional layer D) Submucosal layer

Functional layer

Which term best describes the ovary in this image? A) Polycystic B) Stein-Leventhal C) Hyperstimulated D) Normal follicles E) Dysplastic

Hyperstimulated

What does precocious puberty describe? A) Onset of puberty prior to age 8 B) Delayed puberty later than age 12 C) Isolated breast development prior to age 5 D) Development of pubic hair after development of breast tissue

Onset of puberty prior to age 8

This patient presents with oligomenorrhea, hirsutism, and infertility. The arrows are pointing to: A) Uterus B) Ovaries C) Piriform muscles D) Fallopian tubes E) Bowel

Ovaries

The hormone responsible for inducing ovulation during the normal menstrual cycle is: A) LH B) FSH C) Estrogen D) PAPPA E) Progesterone

LH

In the evaluation of an adnexal mass, the following findings increase the likelihood of malignancy: A) Premenopausal patient with mass that measures less than 6 cm B) Postmenopausal patient with thin-walled, unilocular fluid-filled mass C) Large thick-walled cyst with multiple thick septations and free fluid D) Teenager with complex solid mass showing calcifications E) Symptomatic dilated pelvic vasculature

Large thick-walled cyst with multiple thick septations and free fluid

An early onset of puberty may be the result of a(n): A) renal neoplasm B) ovarian neoplasm C) thyroid gland neoplasm D) pituitary gland neoplasm

ovarian neoplasm

The sonographic findings are most suspicious for which of the following pathologies? A) surface epithelial cysts B) polycystic ovarian disease C) overstimulation syndrome D) normal physiological cysts

polycystic ovarian disease

The section of time previous to the onset of menstruation is termed: A) puberty B) menarche C) premenarche D) perimenopause

premenarche

Which of the following are considered symptoms for endometriosis? A) Menorrhagia and dysmenorrhea B) Dysmenorrhea and dyspareunia C) Menorrhagia, dysmenorrhea, and dyspareunia D) Menorrhagia and cyclic pain E) Menorrhagia, dysmenorrhea, dyspareunia, and cyclic pain

Menorrhagia, dysmenorrhea, dyspareunia, and cyclic pain

Miss Greenfield is 73 years old and asymptomatic. She is not on hormone replacement therapy. Her endometrium should not measure more than: A) 1 mm B) 2 mm C) 3 mm D) 4 mm E) 5 mm

5 mm

What happens to the dominant follicle during the luteal phase? A) It continues to grow until progesterone levels drop, then it regresses. B) It becomes the corpus luteum, produces progesterone, preparing the endometrium for implantation. C) It becomes the corpus albicans in preparation for the next cycle. D) It becomes the corpus luteum, produces estrogen which prepares the endometrium for implantation.

It becomes the corpus luteum, produces progesterone, preparing the endometrium for implantation.

What happens to the corpus luteum if a pregnancy occurs? A) It resolves by the 6th week of pregnancy. B) It can reach a size of up to 10 cm but should resolve by 16 weeks of gestation. C) Production of hCG maintains the corpus luteum throughout the pregnancy. D) It produces estrogen required to maintain the pregnancy

It can reach a size of up to 10 cm but should resolve by 16 weeks of gestation.

Which of the following is not a sonographic finding in pelvic inflammatory disease? A) normal-appearing pelvis B) complex tubular adnexal mass C) focal hypoechoic adnexal mass D) thick and hypervascular endometrium

focal hypoechoic adnexal mass

A 35-year-old patient presents with a history of infertility. Her last menstrual period was 3 weeks previously. A coronal image of the uterus shows a: A) bicornuate uterus B) hypoechoic endometrial mass C) hyperechoic endometrial mass D) normal-appearing endometrial cavity

hyperechoic endometrial mass

Levels of follicle-stimulating hormone begin declining in the: A) late secretory phase B) late follicular phase C) early secretory phase D) early follicular phase

late follicular phase

A large multicystic ovarian mass, in an ovarianstimulated patient, is most suspicious for: A) a corpus luteum B) polycystic ovarian disease C) multicystic ovarian disease D) ovarian hyperstimulation syndrome

ovarian hyperstimulation syndrome

Mittelschmerz is associated with A) pregnancy B) ovulation C) hemorrhage D) menstruation

ovulation

Decreases in estrogen in postmenopausal patients can decrease cervical mucus and can also A) shorten vaginal length B) increase cervical length C) thicken the vaginal walls D) thicken the endometrial cavity

shorten vaginal length

A patient presents with a history of infertility. The sonographic findings in this coronal sonogram are most suspicious for: A) adenomyosis B) subseptous uterus C) bicornuate uterus D) submucosal leiomyoma

subseptous uterus

Which of the following describe the sonographic appearance of the endometrium during the late proliferation phase? A) thick, hyperechoic functional layer and a hyperechoic basal layer B) thin, hyperechoic functional layer and a hypoechoic basal layer C) thick, hyperechoic functional layer and a hypoechoic basal layer D) thick, hypoechoic functional layer and a hyperechoic basal layer

thick, hypoechoic functional layer and a hyperechoic basal layer

What is the average age for climacteric to occur? A) 51 B) 13 C) 58 D) 27

51

What causes the endometrium to become thicker in the first phase of the menstrual cycle? A) Increased levels of follicle-stimulating hormone B) Increased progesterone levels C) Increased estrogen levels D) Increased levels of luteinizing hormone

Increased estrogen levels

DES was prescribed to pregnant women during the 1940s and 1950s to: A) Inhibit spontaneous abortion B) Relieve morning sickness C) Treat trophoblastic disease D) Offset RH isoimmunization E) Relieve pain

Inhibit spontaneous abortion

The term that refers to mid-cycle or ovulatory pain is: A) Dysmenorrhea B) Dyspareunia C) Mittleschmertz D) Menstrual distress E) Premenstrual syndrome

Mittleschmertz

Fitz-Hugh Curtis syndrome consists of: A) Hirsutism and infertility B) Pelvic ascites and pleural effusion C) Webbed neck and gonadal dysgenesis D) Right upper quadrant pain and PID E) Pelvic ascites and metastases to the liver

Right upper quadrant pain and PID

Ovarian volume is lowest during the: A) luteal phase B) ovulatory phase C) menstrual phase D) periovulatory phase

luteal phase

The endometrial echo would appear hypoechoic: A) During the secretory phase of the menstrual cycle B) After dilatation and curettage C) After insertion of an intrauterine device D) During the periovulatory stage E) Upon decidualization

During the periovulatory stage

A 70-year-old patient presented with vaginal bleeding. Ultrasound revealed a focal area of thickening with myometrial distortion and increased vascularity. What is the most likely course of action? A) Follow-up scan in 3 months B) Endometrial biopsy ASAP C) Hysterectomy ASAP D) Dilation and curettage then follow-up in 6 months

Endometrial biopsy ASAP

An 85-year-old female is having postmenopausal bleeding. This longitudinal image suggests A) Fibroids B) Adenomyosis C) Endometrial carcinoma D) Endometrial polyps E) Endometritis

Endometrial carcinoma

A 75-year-old patient presents with vaginal bleeding. Previous ultrasound was negative for any structural abnormalities and she is not taking HRT. Endometrial thickness was 17 mm. What is the most likely cause for the bleeding? A) Endometrial polyps B) Endometrial hyperplasia C) Submucosal leiomyoma D) Endometrial atrophy

Endometrial hyperplasia

A patient with polycystic ovarian syndrome presents with abnormal uterine bleeding. Endometrial measurement is 21 mm and the endometrium demonstrates tiny cystic areas. What is the most likely diagnosis? A) Endometrial hyperplasia B) Endometrial carcinoma C) Endometrial atrophy D) Endometrial polyp

Endometrial hyperplasia

A 52-year-old female presents as postmenopausal for 5 years, G-4-P4. Her uterus is enlarged upon palpation and is irregular in contour. You suspect: A) Endometrial cancer B) Adenomyosis C) Endometriosis D) Hydatidiform mole E) Leiomyoma

Leiomyoma

Which of the following is a possible complication of introduction of intrauterine contraceptive devices? A) Spontaneous abortion B) Ectopic pregnancy C) Perforation of the myometrium D) Asherman syndrome

Perforation of the myometrium

Which phase of the menstrual cycle is this patient in? A) Proliferative B) Secretory C) Periovulatory D) Late menstrual E) Perimenopausal

Periovulatory

A 32-year-old patient presents with a history of endometriosis. Endometriosis is a result of: A) previous pelvic inflammatory disease B) endometrial tissue within the myometrium C) an accumulation of ectopic endometrial tissue D) endometrial tissue within the peritoneal cavity

an accumulation of ectopic endometrial tissue

Estrogen is primarily secreted by the: A) thyroid glands B) corpus luteum C) pituitary gland D) graafian follicle

graafian follicle

Which of the following complications is most likely associated with ovulation induction therapy? A) ectopic pregnancy B) multiple gestations C) spontaneous abortion D) hyperstimulation syndrome

hyperstimulation syndrome

A patient presents with a family history of ovarian carcinoma. The endometrial phase in this patient is most consistent with: A) late secretory B) late menstrual C) early menstrual D) late proliferation

late menstrual

A patient presents with a history of postmenopausal bleeding. A heterogeneous intrauterine mass is identified on sonography. On the basis of the clinical history, the sonographic findings are most suspicious for: A) leiomyoma B) endometrioma C) leiomyosarcoma D) endometrial hyperplasia

leiomyosarcoma

A perimenopausal patient presents with a history of pelvic fullness and pain. A sagittal sonogram displays a fluid collection in the: A) prevesical space B) space of Retzius C) pouch of Douglas D) vesicouterine pouch

pouch of Douglas

A patient who has never had a menstrual period is said to have: A) menopause B) precocious puberty C) primary amenorrhea D) secondary amenorrhea

primary amenorrhea

Which of the following hormones help to prepare the endometrium for implantation of the blastocyst? A) estrogen B) estradiol C) progesterone D) luteinizing hormone

progesterone

In premenarche, the size of the uterine cervix is expected to be: A) half the size of the corpus B) equal to the uterine corpus C) twice as large as the corpus D) equal to the uterine fundus

twice as large as the corpus

Which of the following causes of abnormal uterine bleeding is most likely to result in hypomenorrhea? A) Endometrial polyps B) Hypothyroidism C) Adenomyosis D) Uterine fibroids

Hypothyroidism

The most common procedure for determining if fallopian tubes have been compromised during some event, causing infertility A) Endoscopic surgery B) Sonohysterography C) Fluoroscopy D) Hysterosalpingography E) Endoscopy

Hysterosalpingography

What does the term metrorrhagia refer to? A) Absence of menstrual periods B) Painful and difficult menstruation C) Irregular cycles greater than 35 days apart D) Irregular menstrual bleeding between periods

Irregular menstrual bleeding between periods

The hormone responsible for the final maturation of the follicle and for ovulation is: A) estrogen B) progesterone C) FSH D) LH

LH

The advantages of transvaginal over transabdominal ultrasound when tracking follicular development include all of the following EXCEPT: A) More precise information B) Better resolution C) Greater comfort D) No preparation E) Limited field of view

Limited field of view

Doppler waveforms of the ovarian arterial flow in a premenopausal woman typically show: A) Low-velocity, high-resistance pattern B) High-velocity, low-resistance pattern C) High-velocity, high-resistance pattern D) Low-velocity, low-resistance pattern E) Reverse-flow pattern

Low-velocity, low-resistance pattern

Acute pelvic pain during the periovulatory phase is termed: A) Murphy sign B) Mittelschmerz C) McBurney sign D) Tip of the iceberg

Mittelschmerz

What is the most common cause of secondary amenorrhea? A) Congenital obstruction B) Pregnancy C) Imperforate hymen D) Endocrine disorder

Pregnancy

The hormone responsible for proliferation of the endometrium is: A) estrogen B) progesterone C) FSH D) LH

estrogen

Which of the following ovarian phases coincides with the proliferation phase of the endometrium? A) luteal B) secretory C) follicular D) ovulatory

follicular

Pelvic inflammatory disease is best described as a(n): A) sexually transmitted disease B) specific inflammatory process of the ovaries C) general classification of inflammatory conditions D) specific inflammatory condition of the fallopian tubes

general classification of inflammatory conditions

Visualization of a corpus luteal cyst indicates: A) ovulation is imminent B) ovulation has occurred C) fertilization has occurred D) ovulatory hemorrhage has occurred

ovulation has occurred

Ovarian volume is the highest during the: A) luteal phase B) follicular phase C) menstrual phase D) periovulatory phase

periovulatory phase

Progesterone levels increase in the: A) secretory phase B) follicular phase C) ovulatory phase D) menstrual phase

secretory phase

During ovarian induction therapy, follicles are only measured when exceeding: A) 0.5 cm B) 1.0 cm C) 2.0 cm D) all follicles are measured

1.0 cm

Which of the following is true about Gll- -I ? A) GIFT is gamete intrafallopian transfer B) Both tubes must be normal. C) GIFT has a higher incidence of success than in vitro fertilization (IVF). D) A and B E) A and C

A and C

A 77-year-old nulliparous patient presents with abnormal uterine bleeding. She has a history of taking estrogen-based HRT. Biopsy was performed and demonstrated an endometrial malignancy. What is the most likely diagnosis? A) Adenocarcinoma of the endometrium B) Endometrial hyperplasia C) Granulossa cell tumor D) Stein-Leventhal syndrome

Adenocarcinoma of the endometrium

In the absence of fertilization, the corpus luteum cyst should regress after: A) 4 days B) 8 days C) 10 days D) 14 days

14 days

Fertilization usually occurs ______ hours after ovulation: A) 4-8 B) 8-12 C) 12-24 D) 24-36 E) 36-48

24-36

Miss Crenshaw is 74 years old and is not taking hormones. Her endometrium should not measure more than: A) 1 mm B) 3 mm C) 5 mm D) 6 mm E) 8mm

5 mm

The endometrium is generally thinnest between days: A) 1 to 5 B) 6 to 9 C) 10 to 14 D) 14 to 21

6 to 9

If a 30-year-old female is on day 8 of her menstrual cycle and she has normal regular periods, her endometrium should measure: A) 4 mm B) 6 mm C) 8 mm D) 10 mm E) 12 mm

8 mm

The endometrium of a patient receiving hormone replacement therapy would be considered abnormal if it measured more than: A) 1 mm B) 3 mm C) 5 mm D) 6 mm E) 8 mm

8 mm

Thickness of an asymptomatic postmenopausal endometrium denying hormone replacement therapy should not exceed: A) 2 mm B) 5 mm C) 8 mm D) 10 mm

8 mm

An infertility patient presents with hypomenorrhea. Sonohysterography demonstrates endometrial septations. What is this most indicative of? A) Endometrial polyps B) Asherman syndrome C) Adenomyosis D) Endometritis

Asherman syndrome

This image of the adnexa of a patient at day 20 in her menstrual cycle is consistent with a: A) Follicular cyst B) Corpus luteum cyst C) Dermoid cyst D) Theca lutein cyst E) Neoplastic cyst

Corpus luteum cyst

A patient presents with monthly symptoms of menstruation but does not have any bleeding. What is the term used to describe this condition? A) Amenorrhea B) Cryptomenorrhea C) Hypomenorrhea D) Dysmenorrhea

Cryptomenorrhea

What causes menses to start? A) Drop in estrogen and progesterone B) Increase in estrogen C) Decrease in progesterone D) Increase in follicle-stimulating hormone

Drop in estrogen and progesterone

What is the most likely cause of postmenopausal bleeding when the endometrial thickness measurement is <5 mm? A) Endometrial atrophy B) Endometrial hyperplasia C) Endometrial carcinoma D) Uncontrolled HRT

Endometrial atrophy

What is the main risk associated with estrogen hormone replacement therapy (HRT)? A) Hypertension with associated diabetes B) Thromboembolism C) Endometrial hyperplasia and endometrial carcinoma D) Endometrial atrophy

Endometrial hyperplasia and endometrial carcinoma

It is estimated that 40% of women with the following problem will have trouble conceiving: A) Adenomyosis B) Salpingitis C) Endometriosis D) Endometritis E) Leiomyomatosis

Endometriosis

Which of the following disease processes should NOT be seen in the postmenopausal patient? A) Endometrial carcinoma B) Ovarian cancer C) Endometrial hyperplasia D) Endometriosis E) Leiomyomas

Endometriosis

A patient presents with a history of PID, right upper quadrant (RUQ) pain, fluid in Morison's pouch and elevated liver enzymes. Sonography of the RUQ reveals adhesions between the liver and right hemi-diaphragm. What is the most likely diagnosis? A) Pseudomyxoma peritonei B) Fitz-Hugh-Curtis syndrome C) Hepatitis D) Cholecystitis

Fitz-Hugh-Curtis syndrome

What are the two phases of the ovarian cycle? A) Follicular and proliferative B) Proliferative and secretory C) Follicular and luteal D) Proliferative and luteal

Follicular and luteal

A mature follicle ready for ovulation is referred to as the: A) Corpus luteum B) Graafian follicle C) Corpus albicans D) Preantral follicle E) Theca lutein

Graafian follicle

A 12-year-old patient presents with primary amenorrhea and pelvic pressure. The most likely cause for her symptoms is: A) Pregnancy B) Leiomyoma C) Cystic teratoma D) Pelvic inflammatory disease E) Hematocolpos

Hematocolpos

What is the typical cause of dysfunctional uterine bleeding? A) Hormonal imbalances resulting in endometrial changes B) Idiopathic C) Uterine fibroids D) Cervical polyps

Hormonal imbalances resulting in endometrial changes

The most common complication related to pharmaceutical stimulation of follicular growth is: A) Death B) Failure C) Anovulation D) Hyperstimulation E) Dysmenorrhea

Hyperstimulation

Which of the following endometrial phases demonstrates the thinnest dimension? A) early menstrual B) early secretory C) late proliferation D) early proliferation

early proliferation

If a patient has menstrual cycles that are 17 days apart, what would you describe this as? A) Polymenorrhea B) Oligomenorrhea C) Amenorrhea D) Hypomenorrhea

Polymenorrhea

Which of the following hormone levels can be slightly higher after menopause? A) estrogen B) progesterone C) luteinizing hormone D) follicle-stimulating hormone

follicle-stimulating hormone

A 25-year-old patient presents with an 18-mm anechoic ovarian mass. This is most consistent with a: A) simple cyst B) graafian follicle C) corpus albicans D) serous cystadenoma

graafian follicle

A patient presents with severe right lower quadrant pain for the previous 2 days and a negative pregnancy test. Her last menstrual period was approximately 2 weeks earlier. She denies any history of endometriosis. On the basis of this clinical history, the sonographic finding is most suspicious for a(n): A) cystic teratoma B) graafian follicle C) hemorrhagic cyst D) serous cystadenoma

hemorrhagic cyst

In the menarche patient, the endometrial thickness should not exceed: A) 8 mm B) 10 mm C) 14 mm D) 20 mm

14 mm

The normal menstrual cycle occurs every: A) 20 days B) 24 days C) 28 days D) 32 days E) 40 days

28 days

Which of the following statements is true for postmenopausal patients? A) Endometrial fluid collections are common B) Postmenopausal ovaries frequently exhibit follicles C) Uterine size and echogenicity increase with age D) Endometrial cancer is the most common cause of postmenopausal bleeding E) Hormone replacement therapy decreases one's chances of developing endometrial cancer

Endometrial fluid collections are common

Endometrial proliferation is stimulated by: A) Human choriogonadotropin B) Progesterone C) Testosterone D) Alpha-fetoprotein E) Estrogen

Estrogen

Which of the following presents on ultrasound with an entrance and exit echo? A) Synechiae B) Intrauterine contraceptive device C) Essure device D) Endometrial polyp

Intrauterine contraceptive device

This image demonstrates that the cause of the patient's infertility is: A) Calcified endometrium B) Endometrial agenesis C) Intrauterine device D) Normal endometrial echo appearance E) Endometrial cysts

Intrauterine device

Of the following statements, which is TRUEST for the postmenopausal uterus? A) It decreases in size and is infantile in appearance. B) It becomes hypoechoic and globular C) The endometrium becomes calcific D) The ovaries become more prominent than the uterine corpus. E) It undergoes autohysterectomy

It decreases in size and is infantile in appearance.

The term used to describe the onset of the first menstrual cycle is: A) Dysmenorrhea B) Amenorrhea C) Menopausal D) Menarche E) Menses

Menarche

The normal cessation of menses is called: A) Menopause B) Menarche C) Menstruation D) Mensation E) Mittlepause

Menopause

The ovaries of a patient who has had a bilateral salpingo-oophorectomy will appear: A) Enlarged B) Atrophied C) Nonvisualized D) Prominent but normal E) Normal

Nonvisualized

Fitz-Hugh Curtis syndrome is associated with: A) Pelvic inflammatory disease B) Endometriosis C) Ovarian fibroma D) Polycystic ovaries E) Ovarian cancer

Pelvic inflammatory disease

Which of these patients would not be a good candidate for a transvaginal exam? A) Patient with a possible ectopic pregnancy B) Patient with a threatened abortion C) Patient with an adnexal mass D) Patient with a large fibroid uterus E) Patient who is morbidly obese and postmenopausal

Patient with a large fibroid uterus

This sonogram is typical of: A) Bicornuate uterus B) Polycystic ovaries C) Hypertrophied musculature D) Pelvic inflammatory disease E) Constipation

Polycystic ovaries

The sonographic appearance of the endometrium in this image indicates which phase of the menstrual cycle? A) Preovulatory B) Early gestation C) Postovulatory D) Secretory E) Proliferative

Preovulatory

The failure to ovulate is termed A) Oocyte B) Anovulation C) Amenorrhea D) Ovulatory agenesis E) Ovarian cycle

Anovulation

The most common cause for postmenopausal bleeding is: A) Benign hyperplasia B) Endometrial polyps C) Endometritis D) Endometrial carcinoma E) Cervical cancer

Benign hyperplasia

The drug of choice most commonly used to induce ovulation is: A) hCG B) Pergonal C) Falastem D) FSH E) Clomid

Clomid

This method of birth control is virtually 100% effective in preventing pregnancy: A) Spermicidal foam B) Rhythm method C) Combination oral contraceptives D) Intrauterine contraceptive devices E) Condoms

Combination oral contraceptives

Which of the following best describes a thick-walled cystic mass with internal echoes that was demonstrated on day 20 of the menstrual cycle and resolved by day 5 of the following cycle? A) Follicle B) Follicular retention cyst C) Corpus luteum D) Hemorrhagic cyst

Corpus luteum

Which of the following ovarian tumors would cause precocious puberty in a child? A) Benign cystic teratoma B) Cystadenoma C) Arrhenoblastoma D) Granulosa cell tumor E) Sertoli-Leydig tumor

Granulosa cell tumor

What is the typical sonographic appearance of an endometrium for a patient taking tamoximfen? A) Thickened heterogeneous endometrium with cystic changes B) Thin atrophic endometrium C) Normal appearing endometrium unless endometrial hyperplasia results D) Focal endometrial thickening with cystic changes

Thickened heterogeneous endometrium with cystic changes

What is the sonographic appearance of the proliferative endometrium? A) Thin echogenic line <5 mm B) Three-line sign (echogenic basal layer - hypoechoic functional layer - echogenic canal - hypoechoic functional layer - echogenic basal layer) C) Thick echogenic endometrium >15 mm D) Hypoechoic thin echogenic line 3 to 7 mm

Three-line sign (echogenic basal layer - hypoechoic functional layer - echogenic canal - hypoechoic functional layer - echogenic basal layer)

A patient presented with pelvic pain, infertility, and a palpable right adnexal mass. Sonography demonstrated a multicystic complex right adnexal mass with dirty shadowing and bilateral hydrosalpinx. What is the most likely diagnosis? A) Appendicitis B) Vaginitis C) Tubo-ovarian abscess D) Acute pelvic inflammatory disease

Tubo-ovarian abscess

Which of the following is a risk factor for developing pelvic inflammatory disease (PID)? A) Utilization of and intrauterine contraceptive device B) Pyelonephritis C) Bladder infection D) Late menarche

Utilization of and intrauterine contraceptive device

A procedure involving the transfer of fertilized oocytes into the fallopian tube either laparoscopically or transcervically is: A) GIFT B) IVF C) ZIFT D) DES E) OHS

ZIFT

Which of the following ovarian masses will most likely coincide with this endometrial phase? A) simple cyst B) graafian follicle C) theca lutein cyst D) corpus luteal cyst

corpus luteal cyst

With ovarian induction therapy, intramuscular injection of what hormone triggers ovulation? A) progesterone B) luteinizing hormone C) follicle-stimulating hormone D) human chorionic gonadotropin

human chorionic gonadotropin

Which of the following is a common symptom associated with this PCOS? A) pelvic pain B) dysmenorrhea C) irregular menses D) abdominal distention

irregular menses

Hyperstimulation of the ovaries will likely result in: A) theca lutein cysts B) polycystic disease C) corpus luteal cysts D) hemorrhagic cysts

theca lutein cysts

Based on this sagittal transvaginal image of the uterus, you suspect the patient to be in this phase of the menstrual cycle: A) Proliferative B) Preovulatory C) Periovulatory D) Secretory E) Mid cycle

Secretory

What signals the corpus luteum to continue to produce progesterone when a fertilized egg is approaching the endometrium? A) Gonadotropins released by the sperm cause the corpus luteum to continue to produce progesterone. B) The motion of the cilia in the fallopian tube causes the release of hCG which keeps the corpus luteum producing progesterone. C) The trophoblast cells produce estrogen which maintains the function of the corpus luteum. D) The syncytiotrophoblast cells produce hCG which keeps the corpus luteum producing progesterone

The syncytiotrophoblast cells produce hCG which keeps the corpus luteum producing progesterone

In vitro fertilization: A) Permits the physician to retrieve many oocytes B) Is performed when the tube is not obstructed C) Allows the physician to retrieve only one oocyte D) Decreases the number of embryos to be implanted E) Does not require ultrasound guidance

Permits the physician to retrieve many oocytes

How soon after conception will pregnancy tests produce positive results? A) 1-2 weeks B) 2-3 weeks C) 3-4 weeks D) 4-5 weeks E) 5-6 weeks

1-2 weeks

Until ovulation, ovarian follicles grow at the daily rate of: A) 1-2 mm B) 2-3 mm C) 3-4 mm D) 4-5 mm E) 5-6 mm

2-3 mm

During the ovulatory phase, normal estradiol levels range between: A) 50 and 100 pg/mL B) 10 and 200 pg/mL C) 100 and 200 pg/mL D) 200 and 400 pg/mL

200 and 400 pg/mL

The length of a normal menstrual cycle ranges between: A) 14 and 28 days B) 21 and 28 days C) 21 and 35 days D) 28 and 40 days

21 and 35 days

Which stage of endometriosis involves the ovaries? A) 0 B) 1 C) 2 D) 3

3

The menstrual cycle is influenced by all of the following EXCEPT: A) Hypothalamus B) Pituitary C) Ovaries D) Adrenal E) A and D

Adrenal

The most common cause(s) of infertility among couples is/are: A) Tubal damage B) Endometriosis C) Sperm failure D) Ovulatory failure E) All of the above

All of the above

The term for absence of menses is: A) Amenorrhea B) Dysmenorrhea C) Menorrhagia D) Agenesis E) Metrorrhagia

Amenorrhea

What is a possible complication of salpingitis due to PID? A) Multiple gestations B) Retained products of conception C) Pleural effusion D) Ectopic pregnancy

Ectopic pregnancy

If a postmenopausal patient is asymptomatic and fluid is identified within her endometrial cavity, the most likely cause of the fluid would be: A) Endometrial carcinoma B) Endometritis C) Pelvic inflammatory disease D) Endometrial atrophy E) Vesicovaginal fistula

Endometrial atrophy

What is the most likely explanation for abnormal uterine bleeding with an endometrial measurement of <5 mm in a postmenopausal patient? A) Endometrial hyperplasia B) Endometrial cancer C) Submucosal fibroids D) Endometrial atrophy

Endometrial hyperplasia

A sonohysterogram demonstrated a mass projecting into the endometrium with a defined stalk. What is demonstrated? A) Endometrial polyp B) Submucosal fibroid C) Endometrial carcinoma D) Endometrial diverticulum

Endometrial polyp

Sonohysterography is a common procedure used to determine some causes of infertility, including A) Endometriosis B) Endometrial polyp C) Adenomyosis D) A and B E) All of the above

Endometrial polyp

A patient presents with an elevated CA-125 level, however, upon close examination, no cancers can be demonstrated. What is another cause of an elevated CA-125 level in a perimenopausal woman? A) Endometrial hyperplasia B) Endometrial polyp C) Ovarian fibroma D) Endometriosis

Endometriosis

All of the following could cause pelvic inflammatory disease EXCEPT: A) Sexually transmitted disease B) Ruptured appendix C) TORCH (toxoplasma, rubella, cytomegalovirus, herpes simplex virus) D) Exposure to DES (diethylstilbestrol) E) Tuberculosis

Exposure to DES (diethylstilbestrol)

A patient who has taken oral contraceptives for more than 5 years is at increased risk for developing: A) Renal cancer B) Hepatic adenoma C) Heart disease D) Lung cancer E) Ectopic pregnancy

Hepatic adenoma

Your patient tests positive for chlamydia and presents with severe pelvic pain. This finding is most typical of: A) Hydrosalpinx B) Hematometra C) Ectopic pregnancy D) Ovarian cyst E) Appendicitis

Hydrosalpinx

What is a possible complication of chronic PID? A) Chlamydia B) Appendicitis C) Pyelonephritis D) Infertility

Infertility

Which of these statements is NOT true of endometriosis? A) It is a disease of upper middle-class professional women B) It is more common among Caucasians C) There is a hereditary predisposition D) Symptoms are cyclic. E) It is associated with multiparity

It is associated with multiparity

A 55-year-old patient presents with a 6-month history of pelvic fullness. She has a history of breast cancer and a recent diagnosis of metastatic liver disease. Based on this clinical history, the sonographic findings are most suspicious for: A) endometriomas B) ovarian carcinoma C) pedunculated fibroid D) Krukenberg tumors

Krukenberg tumors

The hormone primarily responsible for ovulation is ________ A) FSH B) LH C) progesterone D) estrogen

LH

What is the cause for increased risk of coronary artery disease and osteoporosis after menopause? A) Lack of circulating estrogen in the blood B) It is coincidental, these risks increase with advanced age C) Decreased progesterone levels in the blood D) Calcium uptake is inhibited by lack of circulating estrogen

Lack of circulating estrogen in the blood

What is the definition of primary amenorrhea? A) Menarche has not occurred by age 16 in a female patient. B) No menstrual cycle in a pregnant patient. C) At least 12 months without a menstrual cycle in a postmenarchal patient. D) Termination of menstrual cycle prior to menopause.

Menarche has not occurred by age 16 in a female patient.

A 49-year-old female patient presents with symptoms of night sweats, dyspareunia, hot flashes, and a decrease in libido. What is the most likely cause? A) Osteroporosis B) Coronary artery disease C) Ovarian teratoma D) Menopause

Menopause

If a 24-year-old female shows a multilayered endometrium measuring 8 mm, she is probably: A) Mid cycle B) Menstruating C) Pregnant D) Menopausal E) Bleeding

Mid cycle

Diethylstilbestrol (DES) is known to increase a patient's risk for all of the following EXCEPT: A) Uterine malformations B) Cervical cancer C) Multiple gestations D) Preterm labor E) Ectopic pregnancy

Multiple gestations

Why is it important to document the type of hormone replacement a patient is taking prior to completing their sonogram? A) Normal endometrial thickness measurements will vary depending on the type of HRT used. B) The number of follicles demonstrated in the ovaries will vary depending on the type of HRT used. C) Uterine size will vary with the type of HRT used. D) Uterine proportions will vary depending on the type of HRT used.

Normal endometrial thickness measurements will vary depending on the type of HRT used.

Stein-Leventhal syndrome is characterized by: A) Menorrhagia, obesity, and hirsutism B) Menorrhagia, obesity, and infertility C) Obesity, hirsutism, and infertility D) Menorrhagia, hirsutism, and infertility E) Menorrhagia, obesity, hirsutism, and infertility

Obesity, hirsutism, and infertility

A patient presents with mittelschmerz. Sonography demonstrated a small amount of free fluid in the right adnexa. What is the likely cause? A) Ovulation B) Ruptured hemorrhagic cyst C) Rupture of nabothian cyst D) Rupture of endometrioma

Ovulation

Which of the following conditions is the most common among pediatric patients? A) Pelvic inflammatory disease B) Endometriosis C) Cervical carcinoma D) Sarcoma botroyoid E) Cystadenoma

Pelvic inflammatory disease

24-year-old patient at day 14 of her menstrual cycle should show a ________ endometrium: A) Proliferative B) Periovulatory C) Secretory D) Decidualized E) Menstrual

Periovulatory

What is the most common indication for performing postmenopausal sonography? A) Postmenopausal bleeding B) Dysmenorrhea C) Palpable pelvic mass D) Urinary frequency

Postmenopausal bleeding

The hormone mostly responsible for premenstrual symptoms and those of early pregnancy is: A) LH B) FSH C) Estrogen D) Progesterone E) PAPPA

Progesterone

In this longitudinal transvaginal image, the appearance of the endometrium is best characterized as: A) Proliferative B) Preovulatory C) Periovulatory D) Secretory E) Decidualized

Proliferative

The phase of the menstrual cycle when the endometrium is its thinnest is the: A) Secretory phase B) Periovulatory phase C) Preovulatory phase D) Luteal phase E) Proliferative phase

Proliferative phase

During which phase of the menstrual cycle is the endometrium at its thickest? A) Menstrual phase B) Secretory phase C) Proliferative phase D) Luteal phase

Secretory phase

The phase of the menstrual cycle following ovulation is referred to as the: A) Proliferative phase B) Periovulatory phase C) Preovulatory phase D) Menstrual phase E) Secretory phase

Secretory phase

Fifty percent of PID cases are caused by which of the following? A) Sexually transmitted diseases B) Abscessed appendix C) Ruptured colonic diverticulum D) Multiple sexual partners

Sexually transmitted diseases

A 30-year-old patient presents with intermenstrual bleeding. Transvaginal ultrasound reveals a mass within the endometrial canal. What test can be used to further differentiate the mass? A) Hysterosalpingogram B) Sonohysterogram with color Doppler C) Endometrial biopsy D) Transrectal sonogram

Sonohysterogram with color Doppler

Which of the following conditions is characterized by amenorrhea, hirsutism, and obesity? A) Stein-Leventhal syndrome B) Endometriosis C) Bicornuate uterus D) Asherman syndrome

Stein-Leventhal syndrome

What happens to the corpus luteum if there is no implantation? A) Luteinizing hormone levels drop resulting in regression of the corpus luteum. B) Luteinizing hormone levels increase causing a decrease in the progesterone levels causing reabsorption of the corpus luteum. C) Progesterone levels drop and result in resorption of the corpus luteum. D) The corpus luteum becomes the corpus albicans due to a decrease in estrogen and progesterone levels.

The corpus luteum becomes the corpus albicans due to a decrease in estrogen and progesterone levels.

Why does menstruation cease at menopause? A) The anterior pituitary gland stops producing follicle-stimulating hormone causing the cycle to stop. B) The ovaries stop producing estrogen and progesterone which halts the cycle. C) The thalamus stops producing luteinizing hormone which halts the cycle. D) Atrophy of the ovaries stops follicles from developing halting the cycle.

The ovaries stop producing estrogen and progesterone which halts the cycle.

Which statement is NOT true of postmenopausal ovaries? A) They should measure less than 2 cm in their greatest dimension B) They should not exceed 8 cubic centimeters in volume each C) They can be recognized by the multiple echogenic foci within them D) They can always be identified transvaginally E) The incidence of ovarian cancer is higher in postmenopausal patients

They can always be identified transvaginally

Why are assisted reproduction patients prone to ovarian hyperstimulation syndrome? A) They have elevated human chorionic gonadotropin levels from ovulation induction. B) Very high levels of estrogen and progesterone induce hyperstimulation of the ovaries. C) Elevated levels of follicle-stimulating hormone cause the ovaries to overproduce follicles enlarging the ovaries. D) Theca lutein cysts occur due to elevated levels of estrogen.

They have elevated human chorionic gonadotropin levels from ovulation induction.

The normal endometrium of a postmenopausal patient is usually: A) Hypoechoic B) Multi-layered C) Thick and hyperechoic D) Thin E) Cystic

Thin

What is luteal phase deficiency? A) Over thickening of the endometrium due to excessive estrogen production B) Underdevelopment of the endometrium due to low progesterone production C) Implantation is prohibited due to fusion of the endometrial canal D) Heterogeneous endometrium produced by high levels of luteinizing hormone

Underdevelopment of the endometrium due to low progesterone production

Which of the following most accurately describes endometriosis? A) proliferation of the endometrial lining B) collection of ectopic endometrial tissue C) ectopic endometrial tissue located in the myometrium D) active endometrial tissue invading the peritoneal cavity

active endometrial tissue invading the peritoneal cavity

When examining a patient referred for "lost IUD string", it would be possible to see which of the following sonographically? A) normal endometrium without evidence of IUD B) uterine perforation by IUD C) IUD properly situated in uterine fundus D) all of the above

all of the above

A 30-year-old patient presents with a history of dysmenorrhea. The arrows in the sonogram are most likely identifying: A) leiomyomas B) adenomyosis C) arcuate vessels D) uterine arteries

arcuate vessels

Assessment for the presence of an ovarian cyst or dominant follicle is scheduled: A) before in vitro fertilization B) after gamete intrafollicular transfer C) before gamete intrafollicular transfer D) before initiating ovarian induction therapy

before initiating ovarian induction therapy

If fertilization does not occur, the corpus luteum will: A) decrease in size and estrogen levels will increase B) increase in size and estrogen levels will decrease C) increase in size and progesterone levels will increase D) decrease in size and progesterone levels will decrease

decrease in size and progesterone levels will decrease

The ovaries in this sonogram coincide with which of the following uterine phases? A) late follicular B) early follicular C) late proliferation D) early proliferation

early proliferation

Which of the following endometrial phases is most likely displayed in this sagittal sonogram of the uterus? A) late secretory B) early menstrual C) early secretory D) early proliferative

early proliferative

An 80 year old patient presents for sonographic evaluation for vaginal bleeding. Which of the following is most important to obtain? A) bilateral ovarian measurements B) transverse uterine diameter C) Doppler evaluation of radial arteries D) endometrial thickness measurement

endometrial thickness measurement

Which of the following is an acquired cause of infertility? A) endometritis B) bicornuate uterus C) Meigs syndrome D) Gartner duct cyst

endometritis

Tamoxifen therapy is most likely to affect which of the following structures? A) cervix B) ovaries C) myometrium D) endometrium

endometrium

A patient presents with a history of intermittent lower quadrant pain. Her last menstrual period was 1 week earlier, and she denies the use of hormone contraceptives. Based on this clinical history, the anechoic areas most likely represent: A) corpus albicans B) functional cysts C) graafian follicles D) corpus luteal cysts

functional cysts

Regeneration of the endometrium occurs as a result of: A) increases in estrogen levels B) decreases in estrogen levels C) increases in progesterone levels D) decreases in progesterone levels

increases in estrogen levels

Which of the following endometrial phases is most likely displayed in the sagittal sonogram? A) early secretory B) late follicular C) early proliferation D) late proliferation

late proliferation

Which ovarian phase has a constant life span? A) luteal B) follicular C) proliferation D) periovulatory

luteal

Which clinical finding is most likely associated with this pathology? A) amenorrhea B) menorrhagia C) dysmenorrhea D) postmenopausal bleeding

menorrhagia

Which of the following complications is commonly associated with in vitro fertilization? A) hyperstimulation B) ectopic pregnancy C) multiple gestations D) spontaneous abortion

multiple gestations

With the gamete intrafollicular transfer technique, the: A) embryos are transferred to the endometrial cavity B) zygotes are transferred to the endometrial cavity C) oocytes and sperm are transferred to the fallopian tube D) oocytes and sperm are transferred to the endometrial cavity

oocytes and sperm are transferred to the fallopian tube

Luteinizing hormone is secreted by the: A) ovary B) hypothalamus C) adrenal gland D) pituitary gland

pituitary gland

Hirsutism is a clinical symptom of: A) endometriosis B) hematometrocolpos C) Asherman syndrome D) polycystic ovarian disease

polycystic ovarian disease

This sagittal image of the uterus most likely represents: A) septae uterus B) bicornuate uterus C) premenarche uterus D) postmenopausal uterus

postmenopausal uterus

If fertilization occurs, the corpus luteum will continue to secrete: A) estrogen B) estradiol C) progesterone D) human chorionic gonadotropin

progesterone

The dominant hormone in the luteal phase of the ovary is: A) progesterone B) estrogen C) FSH D) hCG

progesterone

Days 6-13 in the uterus are considered the _____ phase A) follicular B) luteal C) secretory D) proliferative

proliferative

Which endometrial phase is most likely demonstrated in this endovaginal sonogram? A) luteal B) secretory C) menstrual D) proliferative

secretory

The endometrium displays the greatest thickness during the: A) follicular phase B) secretory phase C) menstrual phase D) proliferation phase

secretory phase

Approximately 15% of postmenopausal patients will exhibit a(n): A) hydrosalpinx B) endometrioma C) simple ovarian cyst D) ovarian malignancy

simple ovarian cyst

Serial sonograms are requested for a patient on Perganol. On day 27 of the cycle, multiple large septated cysts are seen bilaterally. This ovarian finding most likely represents: A) theca lutein cysts B) corpus luteal cysts C) bilateral TOAs D) ectopic pregnancy

theca lutein cysts

The endometrial cavity in patients using hormone contraceptive therapy appears on ultrasound as a: A) thin, echogenic linear structure B) thin, hypoechoic linear structure C) thick, hypoechoic linear structure D) thick, hyperechoic linear structure

thin, echogenic linear structure

hCG is secreted by: A) dividing cells of blastomere B) amnion C) trophoblast/ chorion D) embryo

trophoblast/ chorion


Kaugnay na mga set ng pag-aaral

RHEL - Ch. 9 - CONTROLLING SERVICES AND DAEMONS

View Set

Basic Python Interview Questions

View Set

Earth Day - April 22 地球日 - 4月22日

View Set

UCF - IDS 3933 - Devon Bazata - WWW - Final

View Set

Tema 1: La crisis del Antiguo Régimen, Tema 2: Revoluciones liberales y nacionalismo, Tema 3: La Revolución Industrial y los cambios sociales, Tema 5. Imperialismo, guerra y revolución

View Set